Stay ahead of learning milestones! Enroll in a class over the summer!

G
Topic
First Poster
Last Poster
hard problem
Cobedangiu   7
N an hour ago by arqady
Let $a,b,c>0$ and $a+b+c=3$. Prove that:
$\dfrac{4}{a+b}+\dfrac{4}{b+c}+\dfrac{4}{c+a} \le \dfrac{1}{a}+\dfrac{1}{b}+\dfrac{1}{c}+3$
7 replies
Cobedangiu
Apr 21, 2025
arqady
an hour ago
Bounding number of solutions for floor function equation
Ciobi_   1
N an hour ago by sarjinius
Source: Romania NMO 2025 9.3
Let $n \geq 2$ be a positive integer. Consider the following equation: \[ \{x\}+\{2x\}+ \dots + \{nx\} = \lfloor x \rfloor + \lfloor 2x \rfloor + \dots + \lfloor 2nx \rfloor\]a) For $n=2$, solve the given equation in $\mathbb{R}$.
b) Prove that, for any $n \geq 2$, the equation has at most $2$ real solutions.
1 reply
Ciobi_
Apr 2, 2025
sarjinius
an hour ago
nice system of equations
outback   4
N 2 hours ago by Raj_singh1432
Solve in positive numbers the system

$ x_1+\frac{1}{x_2}=4, x_2+\frac{1}{x_3}=1, x_3+\frac{1}{x_4}=4, ..., x_{99}+\frac{1}{x_{100}}=4, x_{100}+\frac{1}{x_1}=1$
4 replies
outback
Oct 8, 2008
Raj_singh1432
2 hours ago
Two circles, a tangent line and a parallel
Valentin Vornicu   103
N 2 hours ago by zuat.e
Source: IMO 2000, Problem 1, IMO Shortlist 2000, G2
Two circles $ G_1$ and $ G_2$ intersect at two points $ M$ and $ N$. Let $ AB$ be the line tangent to these circles at $ A$ and $ B$, respectively, so that $ M$ lies closer to $ AB$ than $ N$. Let $ CD$ be the line parallel to $ AB$ and passing through the point $ M$, with $ C$ on $ G_1$ and $ D$ on $ G_2$. Lines $ AC$ and $ BD$ meet at $ E$; lines $ AN$ and $ CD$ meet at $ P$; lines $ BN$ and $ CD$ meet at $ Q$. Show that $ EP = EQ$.
103 replies
Valentin Vornicu
Oct 24, 2005
zuat.e
2 hours ago
Inequalities
idomybest   3
N 3 hours ago by damyan
Source: The Interesting Around Technical Analysis Three Variable Inequalities
The problem is in the attachment below.
3 replies
idomybest
Oct 15, 2021
damyan
3 hours ago
Function on positive integers with two inputs
Assassino9931   2
N 3 hours ago by Assassino9931
Source: Bulgaria Winter Competition 2025 Problem 10.4
The function $f: \mathbb{Z}_{>0} \times \mathbb{Z}_{>0} \to \mathbb{Z}_{>0}$ is such that $f(a,b) + f(b,c) = f(ac, b^2) + 1$ for any positive integers $a,b,c$. Assume there exists a positive integer $n$ such that $f(n, m) \leq f(n, m + 1)$ for all positive integers $m$. Determine all possible values of $f(2025, 2025)$.
2 replies
Assassino9931
Jan 27, 2025
Assassino9931
3 hours ago
Normal but good inequality
giangtruong13   4
N 3 hours ago by IceyCold
Source: From a province
Let $a,b,c> 0$ satisfy that $a+b+c=3abc$. Prove that: $$\sum_{cyc} \frac{ab}{3c+ab+abc} \geq \frac{3}{5} $$
4 replies
giangtruong13
Mar 31, 2025
IceyCold
3 hours ago
Tiling rectangle with smaller rectangles.
MarkBcc168   61
N 3 hours ago by YaoAOPS
Source: IMO Shortlist 2017 C1
A rectangle $\mathcal{R}$ with odd integer side lengths is divided into small rectangles with integer side lengths. Prove that there is at least one among the small rectangles whose distances from the four sides of $\mathcal{R}$ are either all odd or all even.

Proposed by Jeck Lim, Singapore
61 replies
MarkBcc168
Jul 10, 2018
YaoAOPS
3 hours ago
A magician has one hundred cards numbered 1 to 100
Valentin Vornicu   49
N 3 hours ago by YaoAOPS
Source: IMO 2000, Problem 4, IMO Shortlist 2000, C1
A magician has one hundred cards numbered 1 to 100. He puts them into three boxes, a red one, a white one and a blue one, so that each box contains at least one card. A member of the audience draws two cards from two different boxes and announces the sum of numbers on those cards. Given this information, the magician locates the box from which no card has been drawn.

How many ways are there to put the cards in the three boxes so that the trick works?
49 replies
Valentin Vornicu
Oct 24, 2005
YaoAOPS
3 hours ago
Nice inequality
sqing   2
N 3 hours ago by Seungjun_Lee
Source: WYX
Let $a_1,a_2,\cdots,a_n  (n\ge 2)$ be real numbers . Prove that : There exist positive integer $k\in \{1,2,\cdots,n\}$ such that $$\sum_{i=1}^{n}\{kx_i\}(1-\{kx_i\})<\frac{n-1}{6}.$$Where $\{x\}=x-\left \lfloor x \right \rfloor.$
2 replies
sqing
Apr 24, 2019
Seungjun_Lee
3 hours ago
Concurrency
Dadgarnia   27
N 3 hours ago by zuat.e
Source: Iranian TST 2020, second exam day 2, problem 4
Let $ABC$ be an isosceles triangle ($AB=AC$) with incenter $I$. Circle $\omega$ passes through $C$ and $I$ and is tangent to $AI$. $\omega$ intersects $AC$ and circumcircle of $ABC$ at $Q$ and $D$, respectively. Let $M$ be the midpoint of $AB$ and $N$ be the midpoint of $CQ$. Prove that $AD$, $MN$ and $BC$ are concurrent.

Proposed by Alireza Dadgarnia
27 replies
Dadgarnia
Mar 12, 2020
zuat.e
3 hours ago
A weird inequality
Eeightqx   0
Mar 29, 2025
For all $a,\,b,\,c>0$, find the maximum $\lambda$ which satisfies
$$\sum_{cyc}a^2(a-2b)(a-\lambda b)\ge 0.$$hint
0 replies
Eeightqx
Mar 29, 2025
0 replies
A weird inequality
G H J
G H BBookmark kLocked kLocked NReply
The post below has been deleted. Click to close.
This post has been deleted. Click here to see post.
Eeightqx
53 posts
#1
Y by
For all $a,\,b,\,c>0$, find the maximum $\lambda$ which satisfies
$$\sum_{cyc}a^2(a-2b)(a-\lambda b)\ge 0.$$hint
Z K Y
N Quick Reply
G
H
=
a